$lim _{xrightarrow c}f(x)=L$ if and only if $lim_{xrightarrow0}f(x+c)=L$












4












$begingroup$



Let $f:= mathbb{R}tomathbb{R}$ and let $cinmathbb{R}$. Show that $lim _{xrightarrow c}f(x)=L$ if and only if $lim_{xrightarrow0}f(x+c)=L$.




From the definition of limit, we get that it is enough to show:

$forall$ $varepsilon>0$ $exists$ $delta>0$ s.t. if $|x-c|<delta$ then $|f(x)-L|<varepsilon$
$Leftrightarrow$ $forall$ $varepsilon_0>0$ $exists$ $delta_0>0$ s.t. if $|x|<delta_0$ then $|f(x+c)-L|<varepsilon_0$




I can replace $x$ by $x+c$ everywhere in statement for the if $(Rightarrow)$ part. But, I am not sure this is the correct method. What I need to do is manipulate the inequalities in each to get the other. But, I am not sure how to proceed with that.










share|cite|improve this question











$endgroup$








  • 4




    $begingroup$
    You mean $lim_{x to c} f(x) = L$ if and only if $lim_{x to 0} f(x+c) = L$.
    $endgroup$
    – Robert Israel
    Dec 19 '18 at 3:03










  • $begingroup$
    @RobertIsrael corrected!
    $endgroup$
    – Za Ira
    Dec 19 '18 at 3:07
















4












$begingroup$



Let $f:= mathbb{R}tomathbb{R}$ and let $cinmathbb{R}$. Show that $lim _{xrightarrow c}f(x)=L$ if and only if $lim_{xrightarrow0}f(x+c)=L$.




From the definition of limit, we get that it is enough to show:

$forall$ $varepsilon>0$ $exists$ $delta>0$ s.t. if $|x-c|<delta$ then $|f(x)-L|<varepsilon$
$Leftrightarrow$ $forall$ $varepsilon_0>0$ $exists$ $delta_0>0$ s.t. if $|x|<delta_0$ then $|f(x+c)-L|<varepsilon_0$




I can replace $x$ by $x+c$ everywhere in statement for the if $(Rightarrow)$ part. But, I am not sure this is the correct method. What I need to do is manipulate the inequalities in each to get the other. But, I am not sure how to proceed with that.










share|cite|improve this question











$endgroup$








  • 4




    $begingroup$
    You mean $lim_{x to c} f(x) = L$ if and only if $lim_{x to 0} f(x+c) = L$.
    $endgroup$
    – Robert Israel
    Dec 19 '18 at 3:03










  • $begingroup$
    @RobertIsrael corrected!
    $endgroup$
    – Za Ira
    Dec 19 '18 at 3:07














4












4








4


1



$begingroup$



Let $f:= mathbb{R}tomathbb{R}$ and let $cinmathbb{R}$. Show that $lim _{xrightarrow c}f(x)=L$ if and only if $lim_{xrightarrow0}f(x+c)=L$.




From the definition of limit, we get that it is enough to show:

$forall$ $varepsilon>0$ $exists$ $delta>0$ s.t. if $|x-c|<delta$ then $|f(x)-L|<varepsilon$
$Leftrightarrow$ $forall$ $varepsilon_0>0$ $exists$ $delta_0>0$ s.t. if $|x|<delta_0$ then $|f(x+c)-L|<varepsilon_0$




I can replace $x$ by $x+c$ everywhere in statement for the if $(Rightarrow)$ part. But, I am not sure this is the correct method. What I need to do is manipulate the inequalities in each to get the other. But, I am not sure how to proceed with that.










share|cite|improve this question











$endgroup$





Let $f:= mathbb{R}tomathbb{R}$ and let $cinmathbb{R}$. Show that $lim _{xrightarrow c}f(x)=L$ if and only if $lim_{xrightarrow0}f(x+c)=L$.




From the definition of limit, we get that it is enough to show:

$forall$ $varepsilon>0$ $exists$ $delta>0$ s.t. if $|x-c|<delta$ then $|f(x)-L|<varepsilon$
$Leftrightarrow$ $forall$ $varepsilon_0>0$ $exists$ $delta_0>0$ s.t. if $|x|<delta_0$ then $|f(x+c)-L|<varepsilon_0$




I can replace $x$ by $x+c$ everywhere in statement for the if $(Rightarrow)$ part. But, I am not sure this is the correct method. What I need to do is manipulate the inequalities in each to get the other. But, I am not sure how to proceed with that.







real-analysis limits analysis






share|cite|improve this question















share|cite|improve this question













share|cite|improve this question




share|cite|improve this question








edited Dec 19 '18 at 3:39









Martin Argerami

129k1184185




129k1184185










asked Dec 19 '18 at 2:57









Za IraZa Ira

161115




161115








  • 4




    $begingroup$
    You mean $lim_{x to c} f(x) = L$ if and only if $lim_{x to 0} f(x+c) = L$.
    $endgroup$
    – Robert Israel
    Dec 19 '18 at 3:03










  • $begingroup$
    @RobertIsrael corrected!
    $endgroup$
    – Za Ira
    Dec 19 '18 at 3:07














  • 4




    $begingroup$
    You mean $lim_{x to c} f(x) = L$ if and only if $lim_{x to 0} f(x+c) = L$.
    $endgroup$
    – Robert Israel
    Dec 19 '18 at 3:03










  • $begingroup$
    @RobertIsrael corrected!
    $endgroup$
    – Za Ira
    Dec 19 '18 at 3:07








4




4




$begingroup$
You mean $lim_{x to c} f(x) = L$ if and only if $lim_{x to 0} f(x+c) = L$.
$endgroup$
– Robert Israel
Dec 19 '18 at 3:03




$begingroup$
You mean $lim_{x to c} f(x) = L$ if and only if $lim_{x to 0} f(x+c) = L$.
$endgroup$
– Robert Israel
Dec 19 '18 at 3:03












$begingroup$
@RobertIsrael corrected!
$endgroup$
– Za Ira
Dec 19 '18 at 3:07




$begingroup$
@RobertIsrael corrected!
$endgroup$
– Za Ira
Dec 19 '18 at 3:07










1 Answer
1






active

oldest

votes


















4












$begingroup$

Given $varepsilon>0$, use the $delta$ from the definition of $lim_{xto c}f(x)=L$. If $|x|<delta$, then $|(x+c)-c|<delta$, so $|f(x+c)-L|<varepsilon$. Thus $lim_{xto0}f(x+c)=L$.






share|cite|improve this answer









$endgroup$













  • $begingroup$
    so, my approach was correct?
    $endgroup$
    – Za Ira
    Dec 19 '18 at 3:41






  • 1




    $begingroup$
    I personally wouldn't be comfortable with "replacing". Your replacement occurs within two quantifiers and inside an implication. I know it works, but if I try to do it your way, it is not easy for me to justify it.
    $endgroup$
    – Martin Argerami
    Dec 19 '18 at 3:43












Your Answer





StackExchange.ifUsing("editor", function () {
return StackExchange.using("mathjaxEditing", function () {
StackExchange.MarkdownEditor.creationCallbacks.add(function (editor, postfix) {
StackExchange.mathjaxEditing.prepareWmdForMathJax(editor, postfix, [["$", "$"], ["\\(","\\)"]]);
});
});
}, "mathjax-editing");

StackExchange.ready(function() {
var channelOptions = {
tags: "".split(" "),
id: "69"
};
initTagRenderer("".split(" "), "".split(" "), channelOptions);

StackExchange.using("externalEditor", function() {
// Have to fire editor after snippets, if snippets enabled
if (StackExchange.settings.snippets.snippetsEnabled) {
StackExchange.using("snippets", function() {
createEditor();
});
}
else {
createEditor();
}
});

function createEditor() {
StackExchange.prepareEditor({
heartbeatType: 'answer',
autoActivateHeartbeat: false,
convertImagesToLinks: true,
noModals: true,
showLowRepImageUploadWarning: true,
reputationToPostImages: 10,
bindNavPrevention: true,
postfix: "",
imageUploader: {
brandingHtml: "Powered by u003ca class="icon-imgur-white" href="https://imgur.com/"u003eu003c/au003e",
contentPolicyHtml: "User contributions licensed under u003ca href="https://creativecommons.org/licenses/by-sa/3.0/"u003ecc by-sa 3.0 with attribution requiredu003c/au003e u003ca href="https://stackoverflow.com/legal/content-policy"u003e(content policy)u003c/au003e",
allowUrls: true
},
noCode: true, onDemand: true,
discardSelector: ".discard-answer"
,immediatelyShowMarkdownHelp:true
});


}
});














draft saved

draft discarded


















StackExchange.ready(
function () {
StackExchange.openid.initPostLogin('.new-post-login', 'https%3a%2f%2fmath.stackexchange.com%2fquestions%2f3045942%2flim-x-rightarrow-cfx-l-if-and-only-if-lim-x-rightarrow0fxc-l%23new-answer', 'question_page');
}
);

Post as a guest















Required, but never shown

























1 Answer
1






active

oldest

votes








1 Answer
1






active

oldest

votes









active

oldest

votes






active

oldest

votes









4












$begingroup$

Given $varepsilon>0$, use the $delta$ from the definition of $lim_{xto c}f(x)=L$. If $|x|<delta$, then $|(x+c)-c|<delta$, so $|f(x+c)-L|<varepsilon$. Thus $lim_{xto0}f(x+c)=L$.






share|cite|improve this answer









$endgroup$













  • $begingroup$
    so, my approach was correct?
    $endgroup$
    – Za Ira
    Dec 19 '18 at 3:41






  • 1




    $begingroup$
    I personally wouldn't be comfortable with "replacing". Your replacement occurs within two quantifiers and inside an implication. I know it works, but if I try to do it your way, it is not easy for me to justify it.
    $endgroup$
    – Martin Argerami
    Dec 19 '18 at 3:43
















4












$begingroup$

Given $varepsilon>0$, use the $delta$ from the definition of $lim_{xto c}f(x)=L$. If $|x|<delta$, then $|(x+c)-c|<delta$, so $|f(x+c)-L|<varepsilon$. Thus $lim_{xto0}f(x+c)=L$.






share|cite|improve this answer









$endgroup$













  • $begingroup$
    so, my approach was correct?
    $endgroup$
    – Za Ira
    Dec 19 '18 at 3:41






  • 1




    $begingroup$
    I personally wouldn't be comfortable with "replacing". Your replacement occurs within two quantifiers and inside an implication. I know it works, but if I try to do it your way, it is not easy for me to justify it.
    $endgroup$
    – Martin Argerami
    Dec 19 '18 at 3:43














4












4








4





$begingroup$

Given $varepsilon>0$, use the $delta$ from the definition of $lim_{xto c}f(x)=L$. If $|x|<delta$, then $|(x+c)-c|<delta$, so $|f(x+c)-L|<varepsilon$. Thus $lim_{xto0}f(x+c)=L$.






share|cite|improve this answer









$endgroup$



Given $varepsilon>0$, use the $delta$ from the definition of $lim_{xto c}f(x)=L$. If $|x|<delta$, then $|(x+c)-c|<delta$, so $|f(x+c)-L|<varepsilon$. Thus $lim_{xto0}f(x+c)=L$.







share|cite|improve this answer












share|cite|improve this answer



share|cite|improve this answer










answered Dec 19 '18 at 3:13









Martin ArgeramiMartin Argerami

129k1184185




129k1184185












  • $begingroup$
    so, my approach was correct?
    $endgroup$
    – Za Ira
    Dec 19 '18 at 3:41






  • 1




    $begingroup$
    I personally wouldn't be comfortable with "replacing". Your replacement occurs within two quantifiers and inside an implication. I know it works, but if I try to do it your way, it is not easy for me to justify it.
    $endgroup$
    – Martin Argerami
    Dec 19 '18 at 3:43


















  • $begingroup$
    so, my approach was correct?
    $endgroup$
    – Za Ira
    Dec 19 '18 at 3:41






  • 1




    $begingroup$
    I personally wouldn't be comfortable with "replacing". Your replacement occurs within two quantifiers and inside an implication. I know it works, but if I try to do it your way, it is not easy for me to justify it.
    $endgroup$
    – Martin Argerami
    Dec 19 '18 at 3:43
















$begingroup$
so, my approach was correct?
$endgroup$
– Za Ira
Dec 19 '18 at 3:41




$begingroup$
so, my approach was correct?
$endgroup$
– Za Ira
Dec 19 '18 at 3:41




1




1




$begingroup$
I personally wouldn't be comfortable with "replacing". Your replacement occurs within two quantifiers and inside an implication. I know it works, but if I try to do it your way, it is not easy for me to justify it.
$endgroup$
– Martin Argerami
Dec 19 '18 at 3:43




$begingroup$
I personally wouldn't be comfortable with "replacing". Your replacement occurs within two quantifiers and inside an implication. I know it works, but if I try to do it your way, it is not easy for me to justify it.
$endgroup$
– Martin Argerami
Dec 19 '18 at 3:43


















draft saved

draft discarded




















































Thanks for contributing an answer to Mathematics Stack Exchange!


  • Please be sure to answer the question. Provide details and share your research!

But avoid



  • Asking for help, clarification, or responding to other answers.

  • Making statements based on opinion; back them up with references or personal experience.


Use MathJax to format equations. MathJax reference.


To learn more, see our tips on writing great answers.




draft saved


draft discarded














StackExchange.ready(
function () {
StackExchange.openid.initPostLogin('.new-post-login', 'https%3a%2f%2fmath.stackexchange.com%2fquestions%2f3045942%2flim-x-rightarrow-cfx-l-if-and-only-if-lim-x-rightarrow0fxc-l%23new-answer', 'question_page');
}
);

Post as a guest















Required, but never shown





















































Required, but never shown














Required, but never shown












Required, but never shown







Required, but never shown

































Required, but never shown














Required, but never shown












Required, but never shown







Required, but never shown







Popular posts from this blog

Plaza Victoria

In PowerPoint, is there a keyboard shortcut for bulleted / numbered list?

How to put 3 figures in Latex with 2 figures side by side and 1 below these side by side images but in...